Những câu hỏi liên quan
Nguyễn Thiều Công Thành
Xem chi tiết
Tuyển Trần Thị
31 tháng 10 2017 lúc 6:13

đúng rồi

Bình luận (0)
Nguyễn Văn Hòa
1 tháng 11 2017 lúc 19:05

 chó điên

Bình luận (0)
bach nhac lam
Xem chi tiết
Ngô Bá Hùng
18 tháng 11 2019 lúc 20:54

1. Vai trò a, b, c như nhau. Không mất tính tổng quát. Giả sử \(a\ge b\ge0\)

\(ab+bc+ca=3\). Do đó \(ab\ge1\)

Ta cần chứng minh rằng \(\frac{1}{1+a^2}+\frac{1}{1+b^2}\ge\frac{2}{1+ab}\left(1\right)\)

\(\frac{2}{1+ab}+\frac{1}{1+c^2}\ge\frac{3}{2}\left(2\right)\)

Thật vậy: \(\left(1\right)\Leftrightarrow\frac{1}{1+a^2}-\frac{1}{1+ab}+\frac{1}{1+b^2}-\frac{1}{1+ab}\ge0\\ \Leftrightarrow\left(ab-a^2\right)\left(1+b^2\right)+\left(ab-b^2\right)\left(1+a^2\right)\ge0\\ \Leftrightarrow\left(a-b\right)\left[-a\left(1+b^2\right)+b\left(1+a^2\right)\right]\ge0\\ \Leftrightarrow\left(a-b\right)^2\left(ab-1\right)\ge0\left(BĐT:đúng\right)\)

\(\left(2\right)\Leftrightarrow c^2+3-ab\ge3abc^2\\ \Leftrightarrow c^2+ca+bc\ge3abc^2\Leftrightarrow a+b+c\ge3abc\)

BĐT đúng, vì \(\left(a+b+c\right)^2>3\left(ab+bc+ca\right)=q\)

\(ab+bc+ca\ge3\sqrt[3]{\left(abc\right)^2}\)

Nên \(a+b+c\ge3\ge3abc\)

Từ (1) và (2) ta có \(\frac{1}{1+a^2}+\frac{1}{1+b^2}+\frac{1}{1+c^2}\ge\frac{3}{2}\)

Dấu ''='' xảy ra \(\Leftrightarrow a=b=c=1\)

Bình luận (0)
 Khách vãng lai đã xóa
Ngô Bá Hùng
18 tháng 11 2019 lúc 21:16

Áp dụng BĐT Cauchy dạng \(\frac{9}{x+y+z}\le\frac{1}{x}+\frac{1}{y}+\frac{1}{z}\), ta được

\(\frac{9}{a+3b+2c}=\frac{1}{a+c+b+c+2b}\le\frac{1}{9}\left(\frac{1}{a+c}+\frac{1}{b+c}+\frac{1}{2b}\right)\)

Do đó ta được

\(\frac{ab}{a+3b+2c}\le\frac{ab}{9}\left(\frac{1}{a+c}+\frac{1}{b+c}+\frac{1}{2b}\right)=\frac{1}{9}\left(\frac{ab}{a+c}+\frac{ab}{b+c}+\frac{a}{2}\right)\)

Hoàn toàn tương tự ta được

\(\frac{bc}{2a+b+3c}\le\frac{1}{9}\left(\frac{bc}{a+b}+\frac{bc}{b+c}+\frac{b}{2}\right);\frac{ac}{3a+2b+c}\le\frac{1}{9}\left(\frac{ac}{a+b}+\frac{ac}{b+c}+\frac{c}{2}\right)\)

Cộng theo vế các BĐT trên ta được

\(\frac{ab}{a+3b+2c}+\frac{bc}{b+3c+2a}+\frac{ca}{c+3a+2b}\le\frac{1}{9}\left(\frac{ac+bc}{a+b}+\frac{ab+ac}{b+c}+\frac{bc+ab}{a+c}+\frac{a+b+c}{2}\right)=\frac{a+b+c}{6}\)Vậy BĐT đc CM

ĐẲng thức xảy ra khi và chỉ khi a = b = c >0

Bình luận (0)
 Khách vãng lai đã xóa
Akai Haruma
18 tháng 11 2019 lúc 22:38

Bài 2:

Áp dụng BĐT AM-GM:

\(a^2+2b^2+c^2=(a^2+b^2)+(a^2+c^2)\geq 2\sqrt{(a^2+b^2)(a^2+c^2)}\geq 2\sqrt{\frac{(a+b)^2}{2}.\frac{(a+c)^2}{2}}=(a+b)(a+c)\)

\(\Rightarrow \frac{ab^2}{a^2+2b^2+c^2}\leq \frac{ab^2}{(a+b)(a+c)}\)

Hoàn toàn tương tự với các phân thức còn lại:

\(\Rightarrow \text{VT}\leq \sum \frac{ab^2}{(a+b)(a+c)}=\frac{a^2b^2+b^2c^2+c^2a^2+abc(a+b+c)}{(a+b)(b+c)(c+a)}\)

Ta cần CM: \(\frac{a^2b^2+b^2c^2+c^2a^2+abc(a+b+c)}{(a+b)(b+c)(c+a)}\leq \frac{a+b+c}{4}\)

\(\Leftrightarrow 4(a^2b^2+b^2c^2+c^2a^2)+4abc(a+b+c)\leq (a+b+c)(a+b)(b+c)(c+a)\)

\(\Leftrightarrow 4(a^2b^2+b^2c^2+c^2a^2)+4abc(a+b+c)\leq (a+b+c)(a+b)(b+c)(c+a)\)

\(\Leftrightarrow 4(a^2b^2+b^2c^2+c^2a^2)+4abc(a+b+c)\leq (a+b+c)[(a+b+c)(ab+bc+ac)-abc]\)

\(\Leftrightarrow 2(a^2b^2+b^2c^2+c^2a^2)\leq (a^3b+ab^3)+(bc^3+b^3c)+(ca^3+c^3a)\)

(dễ thấy luôn đúng do theo BĐT AM-GM)

Do đó ta có đpcm.

Dấu "=" xảy ra khi $a=b=c$

Bình luận (0)
 Khách vãng lai đã xóa
TXT Channel Funfun
Xem chi tiết
bach nhac lam
9 tháng 2 2020 lúc 16:09

\(\frac{a^3}{bc}+\frac{b^3}{ca}=\frac{a^4}{abc}+\frac{b^4}{abc}\ge\frac{\left(a^2+b^2\right)^2}{2abc}\ge\frac{2ab\left(a^2+b^2\right)}{2abc}=\frac{a^2+b^2}{c}\)

Dấu "=" xảy ra \(\Leftrightarrow a=b\)

viết các bđt tương tự rồi cộng vế theo vế là được

Bình luận (0)
 Khách vãng lai đã xóa
Văn Thắng Hồ
Xem chi tiết
Hồng Phúc
15 tháng 10 2020 lúc 22:48

3.

\(5a^2+2ab+2b^2=\left(a^2-2ab+b^2\right)+\left(4a^2+4ab+b^2\right)\)

\(=\left(a-b\right)^2+\left(2a+b\right)^2\ge\left(2a+b\right)^2\)

\(\Rightarrow\sqrt{5a^2+2ab+2b^2}\ge2a+b\)

\(\Rightarrow\frac{1}{\sqrt{5a^2+2ab+2b^2}}\le\frac{1}{2a+b}\)

Tương tự \(\frac{1}{\sqrt{5b^2+2bc+2c^2}}\le\frac{1}{2b+c};\frac{1}{\sqrt{5c^2+2ca+2a^2}}\le\frac{1}{2c+a}\)

\(\Rightarrow P\le\frac{1}{2a+b}+\frac{1}{2b+c}+\frac{1}{2c+a}\)

\(\le\frac{1}{9}\left(\frac{1}{a}+\frac{1}{a}+\frac{1}{b}+\frac{1}{b}+\frac{1}{b}+\frac{1}{c}+\frac{1}{c}+\frac{1}{c}+\frac{1}{a}\right)\)

\(=\frac{1}{3}\left(\frac{1}{a}+\frac{1}{b}+\frac{1}{c}\right)\le\frac{1}{3}.\sqrt{3\left(\frac{1}{a^2}+\frac{1}{b^2}+\frac{1}{c^2}\right)}=\frac{\sqrt{3}}{3}\)

\(\Rightarrow MaxP=\frac{\sqrt{3}}{3}\Leftrightarrow a=b=c=\sqrt{3}\)

Bình luận (0)
 Khách vãng lai đã xóa
Kudo Shinichi
Xem chi tiết
Kudo Shinichi
6 tháng 7 2016 lúc 21:00

Trả lời hộ mình đi

Bình luận (0)
Clary
Xem chi tiết
bach nhac lam
Xem chi tiết
Nguyễn Huy Thắng
19 tháng 11 2019 lúc 20:42

a)\(VT=\sum_{cyc}\frac{ab^3+ab^2c+a^2bc}{\left(a^2+bc+ca\right)\left(b^2+bc+ca\right)}\le\frac{\sum_{cyc}\left(ab^3+ab^2c+a^2bc\right)}{\left(ab+bc+ca\right)^2}\)

\(=\frac{ab^3+bc^3+ca^3+2a^2bc+2ab^2c+2abc^2}{\left(ab+bc+ca\right)^2}\)\(\le\frac{\sum_{cyc}ab\left(a^2+b^2\right)+abc\left(a+b+c\right)}{\left(ab+bc+ca\right)^2}\)

\(=\frac{\left(ab+bc+ca\right)\left(a^2+b^2+c^2\right)}{\left(ab+bc+ca\right)^2}=\frac{a^2+b^2+c^2}{ab+bc+ca}=VP\)

Bình luận (0)
 Khách vãng lai đã xóa
Nguyễn Huy Thắng
19 tháng 11 2019 lúc 20:49

b thiếu đề

Bình luận (0)
 Khách vãng lai đã xóa
bach nhac lam
19 tháng 11 2019 lúc 12:37

@tth_new, @Nguyễn Việt Lâm, @No choice teen, @Akai Haruma

giúp e vs ạ! Cần gấp

Thanks nhiều

Bình luận (0)
 Khách vãng lai đã xóa
hanvu
Xem chi tiết
Trần Phúc Khang
31 tháng 7 2019 lúc 19:53

1. BĐT ban đầu

<=> \(\left(\frac{1}{3}-\frac{b}{a+3b}\right)+\left(\frac{1}{3}-\frac{c}{b+3c}\right)+\left(\frac{1}{3}-\frac{a}{c+3a}\right)\ge\frac{1}{4}\)

<=>\(\frac{a}{a+3b}+\frac{b}{b+3c}+\frac{c}{c+3a}\ge\frac{3}{4}\)

<=> \(\frac{a^2}{a^2+3ab}+\frac{b^2}{b^2+3bc}+\frac{c^2}{c^2+3ac}\ge\frac{3}{4}\)

Áp dụng BĐT buniacoxki dang phân thức 

=> BĐT cần CM

<=> \(\frac{\left(a+b+c\right)^2}{a^2+b^2+c^2+3\left(ab+bc+ac\right)}\ge\frac{3}{4}\)

<=> \(a^2+b^2+c^2\ge ab+bc+ac\)luôn đúng 

=> BĐT được CM

Bình luận (0)
Phùng Minh Quân
31 tháng 7 2019 lúc 21:15

2) \(a+b+c\le ab+bc+ca\le\frac{\left(a+b+c\right)^2}{3}\)\(\Leftrightarrow\)\(\left(a+b+c\right)^2-3\left(a+b+c\right)\ge0\)

\(\Leftrightarrow\)\(\left(a+b+c\right)\left(a+b+c-3\right)\ge0\)\(\Leftrightarrow\)\(a+b+c\ge3\)

ko mất tính tổng quát giả sử \(a\ge b\ge c\)

Có: \(3\le a+b+c\le ab+bc+ca\le3a^2\)\(\Leftrightarrow\)\(3a^2\ge3\)\(\Leftrightarrow\)\(a\ge1\)

=> \(\frac{1}{1+a+b}+\frac{1}{1+b+c}+\frac{1}{1+c+a}\le\frac{3}{1+2a}\le1\)

Dấu "=" xảy ra \(\Leftrightarrow\)\(a=b=c=1\)

Bình luận (0)
Trần Phúc Khang
31 tháng 7 2019 lúc 21:46

Bạn @Diệu Linh@ làm nhầm dòng 5 rồi nhé

2, BĐT ban đầu 

<=> \(\left(1-\frac{1}{1+a+b}\right)+\left(1-\frac{1}{1+b+c}\right)+\left(1-\frac{1}{1+a+c}\right)\ge2\)

<=> \(\frac{\left(a+b\right)^2}{a+b+\left(a+b\right)^2}+\frac{\left(b+c\right)^2}{b+c+\left(b+c\right)^2}+\frac{\left(c+a\right)^2}{c+a+\left(c+a\right)^2}\ge2\)

Dùng BĐT buniacoxki dạng phân thức ở VT 

\(VT\ge\frac{4\left(a+b+c\right)^2}{2\left(a+b+c\right)+\left(a+b\right)^2+\left(b+c\right)^2+\left(c+a\right)^2}\)

Mà \(a+b+c\le ab+bc+ac\)

=> \(VT\ge\frac{4\left(a+b+c\right)^2}{2\left(ab+bc+ac\right)+2\left(a^2+b^2+c^2+ab+bc+ac\right)}=\frac{4\left(a+b+c\right)^2}{2\left(a+b+c\right)^2}=2\)(ĐPCM)

Dấu bằng xảy ra khi a=b=c=1

Bình luận (0)
Fire Sky
Xem chi tiết
tth_new
14 tháng 11 2019 lúc 13:46

\(\Leftrightarrow\Sigma_{cyc}\frac{\left(ab+bc+ca\right)^2}{2a^2+bc}\le\left(a+b+c\right)^2\)

Ta có: \(\frac{\left(ab+bc+ca\right)^2}{2a^2+bc}\le\frac{\left(ab+ca\right)^2}{2a^2}+\frac{\left(bc\right)^2}{bc}=\frac{\left(b+c\right)^2}{2}+bc\)

Tương tự rồi cộng lại ta thu được:

\(L.H.S\le\frac{\left(a+b\right)^2+\left(b+c\right)^2+\left(c+a\right)^2}{2}+ab+bc+ca\)

\(=\frac{2\left(a^2+b^2+c^2\right)+2\left(ab+bc+ca\right)}{2}+ab+bc+ca\)\(=\left(a+b+c\right)^2\)

P/s: Nhìn đơn giản chứ nó là bao nhiêu ngày suy nghĩ đấy ạ:( Chả biết đúng hay sai nữa:v

Bình luận (0)
 Khách vãng lai đã xóa